Questions on Entanglement and Double-Slit Experiment

In summary: Yes, this is correct.2. No, there is no superposition of the results of an unmeasured Stern-Gerlach experiment.
  • #106
ttn said:
Bell's theorem has nothing to do with 'realism', but it does prove that no local theory can agree with experiment. You have a "hunch" that says otherwise... but is this actually based on anything? For example, can you tell us exactly where Bell went wrong in his reasoning?

Bell wasn't wrong and he did acknowledged the limitations of his theory.

In “Speakable and Unspeakable in Quantum Mechanics”, chapter 12, J.S. Bell writes:

It has been argued that quantum mechanics is not locally causal and cannot be embedded in a local causal theory. That conclusion depends on treating certain experimental parameters, typically the orientations of polarization filters, as free variables. Roughly speaking it is supposed that an experimenter is quite free to choose among the various possibilities offered by his equipment. But it might be that this apparent freedom is illusory. Perhaps experimental parameters and experimental results are both consequences, or partially so, of some common hidden mechanism. Then the apparent non-locality could be simulated.

He quite unambiguously states -- and I have studied his work in detail and believe he is right -- that the theorem proves that no local theory can be viable. The argument -- the detailed proof of this conclusion -- is right there in his papers.

Well, it seems you didn't read Bell's work carefully enough or you didn't pay attention to the assumptions he makes. If one assumption falls, the theory falls, regardless of how "detailed" is the "proof of his conclusion".

So the burden is clearly on you to justify your speculation that he was wrong. Otherwise, you're just some schmuck spouting BS on an internet forum.

So, the burden of proof is on you to show that Bell was wrong when he said...

"Perhaps experimental parameters and experimental results are both consequences, or partially so, of some common hidden mechanism. Then the apparent non-locality could be simulated."

...otherwise, you're just some..., oh, forget it.
 
Physics news on Phys.org
  • #107
RandallB said:
Of course Bell doesn’t address deterministic it only address Local vs. Non-local. And deterministic theories are non-local

That's a very strange idea about locality. So, a set of billiard balls, perfectly following Newtons' laws of motion is a non-local system. I can only wonder what do you think it is a local mechanism. An example would be great.

I already gave you Smolin – 3 books and lots of papers; plus READ though the forum below this one (SR & GR) that more than enough to keep you out of trouble till to start to understand enough to talk about GR – take your time and think as you go – remember it took Einstein 10 years – you think you should get it in ten minuets?

Please read my post to ttn and see how I gave him the exact quote from the book that supports my position. I don't have the time to buy and read three books and read everything on the subject on this forum in order to guess what your argument may be. So, please post Smolin's words where he says that GR is non-local.

Plus, what do infinite velocities have to do with anything? Newton was very clear about gravity and he never called for infinite velocity, if someone says he did ask them in which Newton Book, Issue, Chapter, Page and Line he did.

What is the speed of gravity in Newton's theory? What happens with planetary orbits if one assumes a finite speed?

NO we don’t agree because you’re saying that locality could be a correct underlying part of a correct and complete QM – you do not understand Bell-local here.

I think I do.

I agree or I should say QM agrees with me, including DrC IMO, that IF (read it again IF) locality is shown to be true, at that instant anyone that understands QM will admit QM is wrong

I disagree with that. Please, read Bell's own words from my post to ttn.

– The Whole Point Is - the evidence so far is for a Non-Local World.

No, the evidence forces someone to choose between a philosophical assumption, without a shred of scientific evidence (free will) and locality.

But you DO! A classical deterministic is NOT BELL LOCAL.
Bell Local means that as each of the series of individual photons the meet up with the PDC to interact with it there are only the local inputs of the PDC and the one Photon that creates two Photons, only local variables can exist in each of them based on all the conservation of etc. etc. There is nothing to keep the two connected they depart that local area to how ever far away taking only Local Variables with them.
The results found at the later distant test say we don’t know all those variables. Plus even no unknown hidden one we can come up with will explain is the results seen there so far. Thus – if this conclusion is correct then our common sense realistic ideas of reality are wrong; that is the correct reality is non-local and our comfy local realist inclinations are just wrong. Bell can say no more than this – period!
Sure the PDC and each of those photons have a history – BUT to “extend ancient reality” of them to the past reality of any possible measuring devise they may encounter. Even if the have to go all the way back to the big bang, AND expect that “extended reality” to be informative enough as to cause the correlations we see. And sure full and complete determinism would do that but it sure is not BELL LOCAL! And IMO it is a ridiculous hypothesis.

So, everything boils down to your opinion that determinism is a "ridiculous hypothesis". Although you are right that determinism implies the conclusion that everything goes back to the Big-Bang (and possibly beyond it) it may be not necessary to go that far. For example it may be that the PDC is influenced (locally, at the speed of light) by the field generated by the detectors. That's enough, I don't need to take Jupiter's position into the account.

The Non-local BM and MWI theories (and others) with realities that add guide waves and extra dimensions to the “realistic” are far more sensible than any deterministic theory no matter how many dimensions it may or may not have.

I don't debate peoples' opinions. When I'll see an argument I'll answer to it. And you should remember that classical physics is deterministic and didn't look absurd at all to past physicists.

SO PLEASE take this, no free will, deterministic “science” to the Philosophical forums, just use the menu to get there.

Yeah, this is a good advice for you, because you are the one who needs the free-will assumption and refuses to support that assumption with scientific evidence.
 
  • #108
DrChinese said:
2. Realism is simply another word for the completeness doctrine, really as EPR defines it. This can be expressed many different ways as words (and they are just words):

a) Non-realistic = QM is Complete = No greater description of state possible = Observer Dependence.

b) Realistic = QM is Incomplete = Hidden variables = Observer Independent Reality

In the QM formalism, I see this as saying that the state function fundamentally obeys the HUP, so statistical outcomes are dependent on what is observed.

OK, I think that's a dumb way to use the word, but so be it. But the question is: how does this justify the use of the phrase "local realism"? Is your claim that Bell uses some such *assumption* as this "realism" in deriving the inequality? Because that is just wrong. The inequality follows from locality *only*.

Also, it's ludicrous to put "observer dependence" and "observer independent" in your list above, as if these (whatever the heck they are supposed to mean exactly) were somehow equivalent to the other things on the list. If, for example, the wave function alone does provide a complete description of physical states, then this is a fact independent of any "observer". The only sense that this terminology might possibly have is in reference to the fact that, in OQM, a measurement "disturbs" the wave function (the collapse rule). But something like this is true in any theory: if you insert a detector somewhere and let it interact with some system, it's going to affect the state of that system. For example, this is true in Bohm's theory, which you call "realistic".

So, seriously, you need to rethink this terminology. Yes, it's true, virtually everyone around today uses the phrase "local realism" to name the assumptions of Bell and/or the alternatives to OQM. But the mere f act that everybody's doing it, doesn't make it right. In following these unthinking people unthinkingly, you are just buying into their confusions.
 
  • #109
ueit said:
Well, it seems you didn't read Bell's work carefully enough or you didn't pay attention to the assumptions he makes. If one assumption falls, the theory falls, regardless of how "detailed" is the "proof of his conclusion".

Everybody knows about this possibility of super-determinism. But there is a big difference between noting that this is indeed an assumption that's made (in the empirical tests more than the derivation of the theorem, by the way) and asserting that the "assumption falls." I mean, do you actually believe that this assumption is false? That experimenters don't have the freedom to choose what they measure -- or more precisely given the way the experiments are actually done, that there was some kind of systematic correlation between the deliberately uncorrelated random number generators on the two sides of the experiment? Saying "it's possible there's some kind of conspiracy" is completely empty. If you think there's some kind of conspiracy, explain why you think that. Otherwise, the rational response is the same as if you had said "maybe we're all brains in vats" -- namely, unless you can provide at least some shred of evidence to suggest that this is possible, then it's just made up empty BS.



So, the burden of proof is on you to show that Bell was wrong when he said...

"Perhaps experimental parameters and experimental results are both consequences, or partially so, of some common hidden mechanism. Then the apparent non-locality could be simulated."

...otherwise, you're just some..., oh, forget it.


I don't think he was wrong when he said that. I just don't think there is any actual evidentiary basis for believing that there is this kind of conspiracy. And by the way, there is a mountain of positive empirical evidence that this is *not* the case -- namely that we are able to understand a zillion other physical phenomena without any such massive metaphysical conspiracy. Note in particular that one could make this same objection to anything. A "randomized" double-blind drug trial showed that some new drug cures the cancer in all the patients that were given it? So this drug is the cure for cancer! Sweet! Oh, but no, you say that maybe it wasn't double-blind and randomized after all, there was some conspiracy that resulted in all the patients who would have survived *anyway* being just the ones who were "randomly" given the drug, so really the data is based on biased sampling and is invalid. Well, the fact is, in science, unless you can actually provide some kind of empirical basis for this speculation (even if it is minimal) nobody will or should listen to you. And the proof that that's the right response is in the pudding: if people did listen to this kind of arbitrary made up objection, we wouldn't have the drugs we have to cure us of terrible diseases, or any of the other wonderful practical things that science has given us.

Here is another objection you might worry about. Maybe all the people who claim to have done these experiments to test the Bell Inequalities have actually been bought off by the government to lie, in their published papers, about what the results were. It's possible, right? So the burden of proof is on you to prove that this didn't happen if you want to accept Aspect's published paper as really reporting the actual results of his experiments... right?
 
  • #110
ttn said:
OK, I think that's a dumb way to use the word, but so be it. But the question is: how does this justify the use of the phrase "local realism"? Is your claim that Bell uses some such *assumption* as this "realism" in deriving the inequality? Because that is just wrong. The inequality follows from locality *only*.

Well, *you* were the one who asked what realism means. :smile: And I think Einstein's later quote "a particle must have a separate reality independent of the measurements" easily justifies the use of those definitions, including the concept of observer independence. As I said, they are just words - so pick the ones that best describe.

As to the idea that the inequality follows from locality only... that doesn't even fit with your own previous logic (which said that this assumption didn't change the net conclusion when paired with the EPR argument). Clearly, Bell included an assumption involving a more complete state of the system than is present in QM, and this assumption is absolutely critical to his conclusion. I have pointed this assumption out any number of times, and I again quote from the original (where a and b are unit vectors present in QM, but c is not):

(14) ... "It follows that c is another unit vector..." (followed by the math that leads quickly to the inequality)

Bell could have put stars by this to make it easier for the reader to see, but he didn't.
 
  • #111
ueit said:
So, everything boils down to your opinion that determinism is a "ridiculous hypothesis". Although you are right that determinism implies the conclusion that everything goes back to the Big-Bang (and possibly beyond it) it may be not necessary to go that far. For example it may be that the PDC is influenced (locally, at the speed of light) by the field generated by the detectors. That's enough, I don't need to take Jupiter's position into the account.

Um, actually that isn't possible. You can let the detectors affect the particle source all you want and it doesn't matter. The "complete state description" lambda that shows up in Bell's definition of locality is taken after the particles are created and in-flight toward the detectors. So either you have to let the detectors affect the state of the particles *after* they are in-flight (and this means that the detector on one side has to be able to affect the particle on the *other* side, which, the way the experiments are done, requires superluminal / nonlocal causation)... or you have to say that there is some kind of sample-biasing conspiracy whereby the "random" setting of the detector angle *after* the particles have already been emitting, actually isn't random at all, and in particular is correlated with the "random" setting on the far side and also with the state of the particle pair.

So now it's not clear what your objection actually is. Before, in your other post to me, it sounded like you were hung up on the fair sampling issue, and the (arbitrary) possibility that one can't actually test the inequality in an unbiased way, because of some conspiracy of pre-determination in the detector settings. But now it sounds like you think there is a much more serious flaw in Bell's argument -- that there are perfectly local mechanisms whereby the inequality could be violated. If you are worried about this, then you're just wrong and haven't understood the theorem.
 
  • #112
DrChinese said:
Clearly, Bell included an assumption involving a more complete state of the system than is present in QM, and this assumption is absolutely critical to his conclusion.

Sometimes he does and sometimes he doesn't. But, for the millionth time, either way you're still wrong. In the version of the derivation of the inequality that does assume local deterministic hidden variables, Bell explicitly references the EPR argument as proving that such HV's must exist under the assumption of locality. The logic is really simple:

Locality --> HV's (EPR)

then

HV's --> Bell Inequality


So I think even you should be able to recognize that the Inequality follows from the Locality assumption *alone*. You don't need some other distinct extra assumption about 'realism' or whatever. And anyway, it's possible to derive some other versions of a Bell-type inequality (the CHSH in particular) without even going through this two step derivation involving the EPR argument from locality to HV's. You can just start with locality and derive the inequality from that, straight away.

So as I said, either way you're wrong. The inequality follows from locality alone.



I have pointed this assumption out any number of times, and I again quote from the original (where a and b are unit vectors present in QM, but c is not):

(14) ... "It follows that c is another unit vector..." (followed by the math that leads quickly to the inequality)

Bell could have put stars by this to make it easier for the reader to see, but he didn't.

Yes, you always mention this passage, but it just proves you don't understand the paper. The unit vectors a, b, and c are possible directions along which a spin measurement might be made -- they aren't hidden variables or anything like that. And anyway, even if you were right about that, you'd be pointing to the wrong place. What about the "hidden variable" a or b? Those are already not contained in the wave function (I guess), so why are you so obsessed with c? But then, there is no answer to this kind of question since it's based on a fundamental confusion about Bell's paper in the first place.
 
  • #113
ttn said:
Everybody knows about this possibility of super-determinism. But there is a big difference between noting that this is indeed an assumption that's made (in the empirical tests more than the derivation of the theorem, by the way) and asserting that the "assumption falls." I mean, do you actually believe that this assumption is false? That experimenters don't have the freedom to choose what they measure -- or more precisely given the way the experiments are actually done, that there was some kind of systematic correlation between the deliberately uncorrelated random number generators on the two sides of the experiment? Saying "it's possible there's some kind of conspiracy" is completely empty.

Egads, I actually agree with this! LOL. Just to add: extending ueit's thinking, you could actually say that ALL scientific results show behavior that is false, because the results were "predetermined" to support an otherwise unsupportable conclusion.

I will point out this setup to demostrate how difficult ueit's position is:

Suppose the detector settings are determined by separate radioactive samples* (i.e. not an electromagnetic force) which are themselves spacelike separated from both the detector and the entanglement source. Thus the deterministic "first cause" is being propagated through space-time affecting even the splitting of atoms in a sample in such a way as to act in concert with both a distant photon source AND a second distant radioactive source to produce results that match the predictions of QM. Further, the example could be extended to involve dozens of separate radioactive sources, which must now operate in concert.

Whew!

* The radioactive sample appears random, and would be calibrated to have a 50-50 chance of "clicking" within the desired time window. A click would mean setting X, no click would mean a different setting Y. Under the "strict determinism" concept, the presence or absence of a click would also be predetermined, a hypothesis which is not falsifiable.
 
Last edited:
  • #114
I just read your post again and was floored by this statement which I barely noticed before:

DrChinese said:
(where a and b are unit vectors present in QM, but c is not):

(14) ... "It follows that c is another unit vector..." (followed by the math that leads quickly to the inequality)


Seriously, you're totally confused. a and b are unit vectors present in QM? What the hell are you talking about? Where pray tell are these unit vectors in QM? Do you even know what these unit vectors are? It's as if you either hadn't actually read Bell's paper, or had never taken even an undergrad class in QM. There's really no point talking to you about this stuff.
 
  • #115
ttn said:
a and b are unit vectors present in QM? What the hell are you talking about?

You are the one ignoring unit vector c, as you always do. Why won't you address this point? I am saying c is the assumption of realism, one which is not present in the QM formalism and which defies the HUP. If there is no c, there is no Bell inequality. Period. a and b are the setups which match to the predictions of QM AND local realistic theories alike. So a and b are simply common to any theory which purports to describe the results of 2 particle entanglement tests.
 
  • #116
ttn said:
The unit vectors a, b, and c are possible directions along which a spin measurement might be made -- they aren't hidden variables or anything like that.

Well, funny, this is a perfectly good description of a, b and c. The problem is that a and b are testable, a and c are testable, b and c are testable, but a, b, and c together are only hypothetical. THAT IS WHY c IS ASSUMED. How can you be so stubborn about such a simple point? I repeat: QM denies c.

Now please try to derive Bell's Inequality WITHOUT reference to a, b and c simultaneously and show it to us. Then I will gladly accept all of your assertions about locality being the only explicit assumption in Bell.
 
  • #117
DrChinese,

* The radioactive sample appears random, and would be calibrated to have a 50-50 chance of "clicking" within the desired time window. A click would mean setting X, no click would mean a different setting Y. Under the "strict determinism" concept, the presence or absence of a click would also be predetermined, a hypothesis which is not falsifiable.

Question: How do we know these are truly random events VS pseudo random events of very close precision to random?
 
  • #118
Nacho said:
DrChinese,

Question: How do we know these are truly random events VS pseudo random events of very close precision to random?

That is what ueit is saying... that perhaps the events are not truly random. OK, maybe that is so. But that hardly explains why these "pseudo random" events are able to conspire to achieve the Bell test correlations! That is my point.
 
  • #119
ttn said:
Here is another objection you might worry about. Maybe all the people who claim to have done these experiments to test the Bell Inequalities have actually been bought off by the government to lie, in their published papers, about what the results were. It's possible, right? So the burden of proof is on you to prove that this didn't happen if you want to accept Aspect's published paper as really reporting the actual results of his experiments... right?
Likewise, the burden is upon you to show that all reputed journals which published papers by local realists proving that in Aspect's experiments neither a genuine Bell inequality was and could be violated as being in some conspiracy to cover up ``the truth''.

Careful
 
  • #120
ttn said:
First, MWI is not non-local.
? Man you can be impossible to follow when in the very same post you say
ttn said:
Bell's theorem proves that any empirically viable theory has to be nonlocal.
I though you had cleared weeks ago that MWI is not Bell Local or have you decided that MWI cannot provide a viable solution to correlations for some reason?
How can I follow your line of reasoning with what looks to me like a massive contradiction.

"Local realist" is not a theory. Anyone who thinks "realism" is also at play better explain what they mean by that term.
Of course it is a theory, held and explained by both Einstein & John Bell. So strongly held they proposed the concepts behind on experiment that could prove them correct (EPR and Bell Theorem) even though at the time the test could not be performed. Both were willing to put their ideas at risk of being shown wrong by their own experimental ideas!
It was years after Bell was proposed before real experiments could be run, with negative results so far. BUT, but if you read your logic books carefully negative results for local only implies non-local as correct, it is not a positive proof of non-local in precise logic.

And what of the new theories you favor? Have any put together anything of an idea or concept that might someday be put to the test in a real experiment to positively prove their idea?
None that I’ve heard of, IMO the concept of the Local Realist is more completely thought though and understood by Bell and Einstein etc than any if the current on paper theories.

ueit said:
That's a very strange idea about locality. So, a set of billiard balls, perfectly following Newtons' laws of motion is a non-local system. I can only wonder what do you think it is a local mechanism. An example would be great.
? Reread post #97, billiard balls following Newton’s laws is Bell Local. Newton’s Laws do not require complete predestined and predetermined events. Newton does not deny the ability of an unknown ball with an unknown history from entering a local system – complete deterministic theories do, that’s why they demand no free will.
 
  • #121
RandallB said:
And what of the new theories you favor? Have any put together anything of an idea or concept that might someday be put to the test in a real experiment to positively prove their idea?
None that I’ve heard of, IMO the concept of the Local Realist is more completely thought though and understood by Bell and Einstein etc than any if the current on paper theories.

To quote RR's famous line:"There you go again!" You are assuming things based on the incomplete information you have and then extending it into perpetuity. There "are" published papers proposing experiments for quantum entanglement. These papers, including one published by me, and in the APS system, proposed a verifiable experiment to test if energy is being consumed in creating a non-local connection between two separate points in space. It can be proven false as well as well as being proven true so, in the spirit of good science, it says that science can also be incrementally advanced by "eliminating" good ideas that can be proven false, thus eliminating a false path.

What are you doing here except advancing endless theoretical roadblocks to any new knowledge about quantum entanglement, but without putting yourself in any way at risk?

Eric
 
Last edited:
  • #122
DrChinese said:
Well, funny, this is a perfectly good description of a, b and c. The problem is that a and b are testable, a and c are testable, b and c are testable, but a, b, and c together are only hypothetical. THAT IS WHY c IS ASSUMED. How can you be so stubborn about such a simple point? I repeat: QM denies c.

Is your claim that in Bell's derivation he talks about measuring a single particle's spin along 3 directions at once?

By the way, it's not true that according to QM "a and b are testable." You can no more measure the spin of a particle along 2 directions at once, than along 3.

And by the way, it's not even true according to QM that a particle has a definite value of (say) "spin along a", which value is then simply revealed when a measurement is made. So even talking about a *single* spin direction the way Bell talks about them goes beyond QM.

That's why I have no clue why you are so obsessed with "c".

Of course, I repeat for the nth time, don't think that just because Bell is talking about really-existing pre-measurement spin values (along several different directions) that his derivation *assumes* deterministic hidden variables. As he explains, he *derives* these hidden variables from locality (referencing the EPR argument).


Now please try to derive Bell's Inequality WITHOUT reference to a, b and c simultaneously and show it to us. Then I will gladly accept all of your assertions about locality being the only explicit assumption in Bell.

Since the inequality is about correlation coefficients for spin measurements along several different directions, one obviously cannot derive the inequality without ever mentioning said directions. But you need to actually pay attention to what's going on in the derivation, and not just look superficially at what's mentioned. Mentioning 3 different directions -- e.g., talking about the difference in correlation in the a,b type runs vs the a,c type runs -- is *not* the same as assuming that one can measure the spin of a single particle along 3 directions at once, or whatever the heck you think he is assuming.
 
  • #123
RandallB said:
? Man you can be impossible to follow when in the very same post you say

I though you had cleared weeks ago that MWI is not Bell Local or have you decided that MWI cannot provide a viable solution to correlations for some reason?
How can I follow your line of reasoning with what looks to me like a massive contradiction.

I don't remember ever saying that MWI was local. Assessing it clearly can be tricky, though, since the theory itself is so outrageously vague (e.g., there are so many radically different versions of it). The theory is local in at least one sense: the dynamical equations that define the theory are fully Lorentz invariant (well, leaving aside the fact that in some version we are missing still the dynamical equations for consciousness, and so there's no way to say for sure). And there's a sense in which it isn't Bell Local, but only the sense that you can't really apply Bell Locality to the theory at all. Remember, Bell Locality is defined in terms of whether or not the probabilities for various particular outcomes depend (according to the theory whose Bell Locality you're trying to assess) on spacelike separated events. Well, in MWI, "particular outcomes" simply don't happen. And there aren't really any distinct "spacelike separated events" either, which might or might not affect the probability of those particular outcomes (which don't happen). So is MWI Bell Local? Not really, but more because the theory just radically fails to posit particular events happening on a spacetime background such that it even makes any sense to talk about whether various events affect each other or not -- than because it cleanly violates Bell's locality criterion. In short, MWI is so radically at odds with the relativistic picture of the world, that it's barely possible to point to anyone particular feature of the theory which renders it inconsistent with relativity.

There's also the very problematic fact that MWI requires us to believe that everything we believe is a delusion. So in a way, even if one grants that it is a local theory, it still isn't any kind of counterexample to the claim that Bell's Theorem proves that no local theory can be empirically viable. For what it means to be "empirically viable" is to agree with the outcomes of the experiments that have been done. But MWI doesn't. Rather, it asserts that the *actual* outcome of (e.g.) Aspect's experiment was completely and totally different than what Aspect himself believed, what he subsequently wrote up in the paper, what appears in the printed copies of the journals, etc. Remember that the reported data from that experiment involves calculating correlation coefficients for photon polarization measurements, each of which had some particular outcome recorded by a computer -- at least so Aspect and we came to think. But according to MWI, that isn't what happened at all, and so all of our beliefs about it (including in particular the belief that the experiment actually displayed a violation of Bell's inequality) are fantasies, delusions. And so, in this sense, whether MWI is local or not is totally irrelevant. It requires us to completely abandon the normal way of doing science -- namely, basing our beliefs on the outcomes of experiments -- because it requires us to believe that the real way the world is (including facts about what outcome happened in a given run of Aspect's experiment or which way some pointer is pointing) radically fails to correspond to our beliefs about it.


Of course [local realism] is a theory, held and explained by both Einstein & John Bell.

I'm not going to waste time arguing about how you use words, but... would you say "determinism" is a theory? There might be theories that are deterministic, sure, but "determinism" is not a theory. Likewise, "locality" is not a theory. And as soon as someone explains what "realism" means and why it has any relevance to this discussion, I'm sure I'll want to say that "local realism" is not a theory either.


So strongly held they proposed the concepts behind on experiment that could prove them correct (EPR and Bell Theorem) even though at the time the test could not be performed. Both were willing to put their ideas at risk of being shown wrong by their own experimental ideas!
It was years after Bell was proposed before real experiments could be run, with negative results so far. BUT, but if you read your logic books carefully negative results for local only implies non-local as correct, it is not a positive proof of non-local in precise logic.

Uh, you'll have to elaborate that last part, because it sounds like you're saying that according to logic, "not A" doesn't necessarily mean that A is false.



And what of the new theories you favor? Have any put together anything of an idea or concept that might someday be put to the test in a real experiment to positively prove their idea?

Bell was surprised by the nonlocality of Bohm's theory (which he liked because it refuted all the bogus "proofs" that such a theory couldn't exist) and found a way to test whether nonlocality is a real fact about nature, or not. How about that?
 
  • #124
ttn said:
Bell was surprised by the nonlocality of Bohm's theory (which he liked because it refuted all the bogus "proofs" that such a theory couldn't exist) and found a way to test whether nonlocality is a real fact about nature, or not. How about that?
How about that for what??; Some kind of contrived revisionist false view of history?
Bell wasn’t betting on finding or expecting a non-local solution, he was building a theorem he hoped could drive discovery of a HVT local solution. Mostly because he recognized the then accepted proof against EPR by John von Neumann as mathematically silly. Even so, he did not get his wish.

And exactly what are a few of these “bogus proofs" that claimed a non-local solution like BM etc could not be described? Not Bell and not Von Neumann -- what?
 
Last edited:
  • #125
I think there was as typo in my last post. I meant to say "I don't remember ever saying that MWI was *non-local*"... not "I don't remember ever saying that MWI was *local*". I would have thought it would have been obvious from the context that this was just a typo, yet you jump on me as follows:

RandallB said:
Then as I’d quoted you form post #101 what could your claim
“First, MWI i s not non-local.”
possibly mean?
Is there some way a “non non-local” can be something other than local?

It means the same thing I meant to say above -- that the problem with MWI is not that it is non-local, but something else. Let me repeat: where exactly did I give you the impression that I thought MWI was a nonlocal theory?

Or just forget it. The detailed discussion in my previous response to you contains as much as I'm interested in saying about MWI.



How about that for what??; Some kind of contrived revisionist false view of history?

So you're denying that Bell was motivated by the manifest non-locality of Bohm's Theory to see whether a hidden variable type theory (which agreed with the predictions of OQM/Bohm, but which was *local*) could exist?

You obviously haven't read Bell's papers (very carefully, or maybe at all) since he says this repeatedly. Or maybe you think he was lying?


Bell wasn’t betting on finding or expecting a non-local solution, he was building a theorem he hoped could drive discovery of a HVT local solution.

That's probably true. What sane person wouldn't have thought (based on the success of relativity theory) that the ultimate theory should be local? But I don't understand why you think this contradicts what I said before. Bohm's theory showed Bell that the "impossibility proofs" were all bogus, which rekindled his interest in the possibility of a more sensible alternative to the Copenhagen theory. But Bohm's theory manifested a blatant nonlocality. So he wondered if a local HVT could exist... or, pretty much equivalently, he wondered if the arbitrary criteria used in all the bogus impossibility proofs, could be replaced by a reasonable assumption about local causality. Don't you see that this is just a particular way of addressing the original question (whether a local HVT could exist)?



And exactly what are a few of these “bogus proofs" that claimed a non-local solution like BM etc could not be described? Not Bell and not Von Neumann -- what?

Well, Gleason and Kochen/Specker are the most famous ones. You should try reading Bell's papers on this stuff, and maybe some of the rest of the literature too while you're at it.
 
  • #126
ttn said:
I think there was as typo in my last post. I meant to say "I don't remember ever saying that MWI was *non-local*"... not "I don't remember ever saying that MWI was *local*". I would have thought it would have been obvious from the context that this was just a typo, yet you jump on me as follows:
No it is not obvious at all as your making less sense than your prior agruments
- but you should still have time to edit your post with the typo
- I have already edited it out of my post so it will not be confusing to others

EDIT : update on the MWI issue alone;
OK so MWI so if MWI is something “other than non-local” that only leaves you with having always declared it to be Local – fine.

But then you still have a contradiction when you agree that only non-local theories can resolve entanglement and then proceed to show how MWI does exactly that. I agree that the MWI solution is hard to follow and based on the definitions of MWI may mean, as you say, only exist in our minds. But don’t you see all that rigmarole is not within the definition of a Bell Local Attributes – it is something outside those Bell local limits and therefore cannot be considered Bell Local. MWI is a non-local theory.
 
Last edited:
  • #127
DrChinese said:
Well, funny, this is a perfectly good description of a, b and c. The problem is that a and b are testable, a and c are testable, b and c are testable, but a, b, and c together are only hypothetical. THAT IS WHY c IS ASSUMED. How can you be so stubborn about such a simple point? I repeat: QM denies c.

Now please try to derive Bell's Inequality WITHOUT reference to a, b and c simultaneously and show it to us. Then I will gladly accept all of your assertions about locality being the only explicit assumption in Bell.

DrChinese, I'd welcome your clarifying and other comments on this:

QM denies c? I think not. But I'm sure that we agree that a, b, c are NOT simultaneously testable (whether we be realists or something other).

Now Bell begins with a, b and c testable in pairs; and concludes with a, b and c in pairs.

So if you examine where Bell uses abc simultaneously*, you will have found an error that we can agree upon. Yes? (And there must be at least one error because his theory does not hold in general. Yes?)

* [See the two un-numbered equations at the top of Bell (1964: 198).] QED?
 
Last edited:
  • #128
ttn said:
Is your claim that in Bell's derivation he talks about measuring a single particle's spin along 3 directions at once?

By the way, it's not true that according to QM "a and b are testable." You can no more measure the spin of a particle along 2 directions at once, than along 3.

And by the way, it's not even true according to QM that a particle has a definite value of (say) "spin along a", which value is then simply revealed when a measurement is made. So even talking about a *single* spin direction the way Bell talks about them goes beyond QM.

That's why I have no clue why you are so obsessed with "c".
Since the inequality is about correlation coefficients for spin measurements along several different directions, one obviously cannot derive the inequality without ever mentioning said directions. But you need to actually pay attention to what's going on in the derivation, and not just look superficially at what's mentioned. Mentioning 3 different directions -- e.g., talking about the difference in correlation in the a,b type runs vs the a,c type runs -- is *not* the same as assuming that one can measure the spin of a single particle along 3 directions at once, or whatever the heck you think he is assuming.

I'll go slow. 2 entangled particles can be measured separately in any 2 directions, let's call them a and b. QM says that there is a statistical relationship between the expected outcomes. That is the end of the story for QM.

But ANY realistic theory postulates that there are simultaneous values for any other set of directions, c/d/e... whatever. The idea is that those values are real and definite INDEPENDENT of the ACT of OBSERVATION. So let's focus on c, that is enough to get the idea across. Prepare an equation which shows the relationship between a, b and c and you will immediately run into the Bell Inequality. All you need to do is prepare a table of possible result values for a, b and c SIMULTANEOUSLY and you will discover - as Bell did - that there are negative probabilty values at certain angle settings.

Any questions?
 
  • #129
wm said:
DrChinese, I'd welcome your clarifying and other comments on this:

QM denies c? I think not. But I'm sure that we agree that a, b, c are NOT simultaneously testable (whether we be realists or something other).

Now Bell begins with a, b and c testable in pairs; and concludes with a, b and c in pairs.

No, the entire idea is that you assume there is an a, b and c simultaneously - if you are a realist. No realist - I think I can speak for this point of view reasonably - is going to say that c doesn't exist just because it can't be measured along with a and b! Keep in mind: Einstein said that particles have attributes even when they are not being observed. If you believe this, then you run afoul of Bell's Theorem. Unless, of course, you believe that there are non-local forces - which is ttn's argument.

I am sorry that this point has not been better explained in the literature, but I assumed that anyone who followed Bell's argument in its mathematical detail would catch this. You must follow his argument from (14) through (16) to see this is what he is saying. I have a page on my site which details the Negative Probabilites issue more specifically, which shows the result table for a, b and c. Follow that link to part b. for more.
 
Last edited:
  • #130
DrChinese said:
I'll go slow.

Thanks, that's helpful, because there are SOOOOOOOO many errors packed into your 2 short paragraphs. If you went faster, I'd no doubt miss some of them.


2 entangled particles can be measured separately in any 2 directions, let's call them a and b. QM says that there is a statistical relationship between the expected outcomes. That is the end of the story for QM.

All right.


But ANY realistic theory postulates that there are simultaneous values for any other set of directions, c/d/e... whatever.

Bohm's theory doesn't postulate that. So it isn't a realistic theory by your definition of that term? This just goes to show that your definition of that term is pointless/misleading/vague/sloppy.


The idea is that those values are real and definite INDEPENDENT of the ACT of OBSERVATION. So let's focus on c, that is enough to get the idea across. Prepare an equation which shows the relationship between a, b and c and you will immediately run into the Bell Inequality.

You have to be more careful. Here's an equation that involves all three. Say, for electrons whose spin is completely random (coming out of an oven or something, so the spin part of the density matrix is proportional to the identity matrix)

P(a) + P(b) + P(c) = 3/2

where P(a) = the probability for the electron to be measured as spin-up along the a-direction if that measurement is made. This is a perfectly well defined and valid equation even in the context of orthodox QM. My point is just that you can't point to an equation that involves these 3 variables and infer *on that basis alone* that there is something unphysical or untoward going on, such as a presumption that 3 incompatible measurements all happen at once. There is no such presumption in the above equation, and also none in Bell's equations that you always point to.

That said, it is true that those equations you point to in Bell's paper do not pertain to orthodox QM. They contain, after all, functions like A(a,lambda) which denote the outcome that would obtain if a measurement is made along direction "a" when the particles are in state lambda. (Somewhat more precisely, this denotes not the actual outcome of any actual experiment, but the prediction of the deterministic theory in question here for such an experiment.) This is what leaves me so confused about your obsession with "c". In Bell's paper, we have

P(a,b)

and

P(a,c)

as the correlations when the pair (a,b) is measured and when the pair (a,c) is measured. If Bell started talking about P(a,b,c) -- the correlation when all three quantities are measured at once -- then I guess you'd have him, since it's not possible to measure all 3 at once. But he doesn't, so what the heck is your problem? What are you objecting to? Surely it's perfectly sensible for a theory to make some kind of prediction for P(a,b). And surely it's sensible for a theory to make some kind of prediction for P(a,c). And surely it's sensible to take those two numbers and subtract them... just like, in OQM in my example above, there is a well-defined quantity P(a) = 1/2 and another well defined quantity P(b) = 1/2, and another well-defined quantity P(c) = 1/2, and once you have those, there's nothing the slightest bit illegal about adding those up to find 3/2. Of course, if you are mentally challenged and you think that this means "when you do a magic experiment that measures all 3 quantities at once, your experiment will have outcome 3/2" then, sure, that's a problem. But it's your problem, not the problem of the theory. And likewise with Bell's consideration of P(a,b) - P(a,c).

And to repeat what I said before, why in the world do you only start fussing about "c", when already Bell has defined A(a,lambda) in a way that is completely at odds with orthodox QM? I mean, once you allow *that*, then there is no valid way to object to anything that happens later (as I tried to explain in the last paragraph). In OQM, there is no such function as A(a,lambda), right? First off, OQM isn't deterministic, so the idea of there being a *function* which specifies *the outcome* for a given setting, is clearly at odds with OQM. Furthermore, in OQM, specifying "a" and the state of the pair (lambda = the 2 particle wf for OQM) is *not* enough information even to predict the *probabilities* of possible outcomes. The theory is nonlocal in the sense that you must specify also some information pertaining to bob's experiment in order to define (even) the probabilities for Alice's. So in at least these two ways, the function

A(a,lambda)

is at odds with OQM. So if you're going to object to something in Bell's derivation, it should be *that*.

But of course, as I've also explained to you a million times, that objection isn't going to work, because Bell has a *reason* for making this un-orthodox assumption of functions like A(a,lambda). He describes it quite clearly in the opening paragraph of his paper when he says:

"The paradox of [EPR] was advanced as an argument that [QM] could not be a complete theory but should be supplemented by additional variables. These additional variables were to restore to the theory causality and locality." He doesn't rehearse the argument here, but the point is that he is citing EPR as proving that the "hidden variables" -- in particular, some variables which locally determine the outcomes once the state of the pair (lambda) and the local setting (a) are specified, i.e., just exactly the functions A(a,lambda) he uses later in the paper -- must exist *in order to restore locality*, which means: in order to replace the *nonlocal* theory OQM with something that is local. In short, he cites EPR as showing that Locality requires exactly the kind of un-orthodox HV's he uses later in the paper. Got it?

Of course, the conclusion of the paper is that this strategy fails. You can't get the empirically correct predictions with those local deterministic hidden variables... which means that, since those variables were required in the first place as the only possible way of saving locality, locality cannot be saved... which Bell says quite clearly in his opening paragraph: "It is the requirement of locality, or more precisely that the restult of a measurement on one system be unaffected by operations on a distant system with which it has interacted in the past, that creates the essential difficulty." He also mentions Bohm's theory as a counterexample to a bunch of earlier (bogus) impossibility proofs, and notes that Bohm's theory "has indeed a grossly nonlocal structure." He then restates what the main conclusion of his paper will be: "This [the nonlocal structure] is characteristic, according to the result to be proved here, of any such theory which reproduces exactly the quantum mechanical predictions."

So to summarize:

1. Your objection about "c" makes no sense

2. If you are going to object to something, you should object earlier -- to Bell's equation (1)

3. But don't bother, because the answer to that objection is the EPR argument as discussed quite clearly by Bell in this paper and many others.
 
  • #131
tnn
I don’t care to spend much time on MWI either and edited my comments on that into post #126.

Sure Bell considered BM worth looking at, even an inspiration. That does not mean the Bell Theorem is designed to confirm a non-local theory like BM.
EPR and “Bell Local” call for particles and entities like photons to have and retain exact specific attributes even when they are not being observed. These would be the Hidden Variables describable in common sense 3 dimensional terms, carrying with them enough independent information to account for correlations. Of course the problem is no such description has ever been put forward – hence Einstein described them as unknown HV’s.
If you read all of John Bell you will see even in his last few years it was clear his intent was not to confirm a non-local theory like BM. As he always expressed disappointment that his theorem did not make known those unknown variables and that he still hoped that maybe they could be found, but he was very clear to point out that it seemed very likely that HVs could not exist, even if he wished that they did.
So stop implying that Bell was some sort of champion for the cause of BM.

But, I finally think I understand the conflict you have with DrC.
You seem to think you can use the Bell Theorem as an argument for BM over other non-local theories. That doesn’t fly, Bell simply cannot do that, it only addresses Bell Local vs. Non Bell Local.
The test is not what you think it proves, if the Bell Theorem provided acceptable creditable scientific proof for BM then others like DrC would already be converting to it. They are not because it does not.

Something new, predicting something new, will be required of any non-local that wishes to prove itself a better solution than other non-locals.
 
  • #132
RandallB said:
tnn
Sure Bell considered BM worth looking at, even an inspiration. That does not mean the Bell Theorem is designed to confirm a non-local theory like BM.

I didn't mean to claim that Bell's Theorem is designed to confirm BM specifically -- only that it was designed to test (and does in fact turn out to confirm) a particular *feature* of BM, namely non-locality.



So stop implying that Bell was some sort of champion for the cause of BM.

I don't know when I did imply that. But if I did, so be it, because it's true. See, for example, the end of his beautiful essay on "6 possible worlds..."


But, I finally think I understand the conflict you have with DrC.
You seem to think you can use the Bell Theorem as an argument for BM over other non-local theories.

No, I don't think that. I think I can use the Bell Theorem as an argument that all viable theories have to be nonlocal. This only supports BM in an indirect way -- by showing that one of the main reasons people have given *against* Bohm's theory (namely, that it is nonlocal) is not a valid reason (since there is no viable alternative which *is* local, i.e., since *nature* is non-local).


The test is not what you think it proves, if the Bell Theorem provided acceptable creditable scientific proof for BM then others like DrC would already be converting to it. They are not because it does not.

You misunderstand the argument I'm making. Hopefully the above clarifies. But, in any case, the reason Dr C and I infer different things from Bell's Theorem is because one of us understands the theorem, and one doesn't. And the mere fact that someone misunderstands something is never a particularly good argument that that something isn't valid or doesn't have the implications someone else says it does.



Something new, predicting something new, will be required of any non-local that wishes to prove itself a better solution than other non-locals.

I don't disagree. My problem is with people who fail to see the battle as among several non-local theories, e.g., those who think that it's only "hidden variable theories" or "realist theories" or "deterministic theories" which are plagued by nonlocality.
 
  • #133
ttn said:
But, in any case, the reason Dr C and I infer different things from Bell's Theorem is because one of us understands the theorem, and one doesn't. And the mere fact that someone misunderstands something is never a particularly good argument that that something isn't valid or doesn't have the implications someone else says it does.

Randallb said:
Something new, predicting something new, will be required of any non-local that wishes to prove itself a better solution than other non-locals.

I don't disagree. My problem is with people who fail to see the battle as among several non-local theories, e.g., those who think that it's only "hidden variable theories" or "realist theories" or "deterministic theories" which are plagued by nonlocality.
Fair enough, I think I at least understand where you’re at. Maybe I'm wrong about Dr C, but unless I missed where he plainly declared BM as being local which would not be correct, I find no fault with his understanding of the Bell Theorem. I’ll just leave it that I do not see the detail in your disputes that justify the abusiveness in some of your disagreements and it may well be beyond me.

I do agree that in science the MAIN battle is between among several non-local theories, But, I do feel it is right and proper that at least a skirmish of a fight should continue in the search of HVT.
But in that fight I too seem to be at odds with most other “Local Realist” when they wish to simply reject Bell, and I cannot. And for me that means "deterministic theories" and MWI are non-local.
 
Last edited:
  • #134
ttn said:
That said, it is true that those equations you point to in Bell's paper do not pertain to orthodox QM. ... In Bell's paper, we have

P(a,b)

and

P(a,c)

as the correlations when the pair (a,b) is measured and when the pair (a,c) is measured. If Bell started talking about P(a,b,c) -- the correlation when all three quantities are measured at once -- then I guess you'd have him, since it's not possible to measure all 3 at once. But he doesn't, so what the heck is your problem? What are you objecting to? Surely it's perfectly sensible for a theory to make some kind of prediction for P(a,b). And surely it's sensible for a theory to make some kind of prediction for P(a,c).

Of course the realistic position is that P(a,c), P(a,b), and P(b,c) don't change according to your choice of which of these you choose to measure. This is basic, and should be uncontrovertible.

And of course there is the assumption that:

1 >=P(a, b, c) >=0

which is also the realistic position restated. Who really cares if P(a, b) and P(a, c) can be measured separately UNLESS measurements at a, b and c all had simultaneous definite outcomes? Unless, as you point out, there is non-local communication occurring which skews the results...
 
  • #135
ttn said:
Everybody knows about this possibility of super-determinism. But there is a big difference between noting that this is indeed an assumption that's made (in the empirical tests more than the derivation of the theorem, by the way) and asserting that the "assumption falls."

This is not about empirical tests. The validity of the theorem itself depends on it. If the detector orientations are not independent variables there is no Bell theorem. I'm not saying that the assumption is necessarily false in our universe (it might be that we have free will after all) but it is incompatible with the assumption of a deterministic HV theory underlying QM. In order to avoid a useless circular reasoning you have to drop this assumption when discussing deterministic theories therefore you cannot use Bell's theorem to dismiss such theories.

I mean, do you actually believe that this assumption is false?

If the universe is deterministic this is a logically inescapable conclusion.

That experimenters don't have the freedom to choose what they measure -- or more precisely given the way the experiments are actually done, that there was some kind of systematic correlation between the deliberately uncorrelated random number generators on the two sides of the experiment?

I think that the particles are generated at the source in accordance with the surrounding fields (EM, gravitational, weak, whatever). I think that the "choice" to do a certain measurement is a result of the same fields.

Saying "it's possible there's some kind of conspiracy" is completely empty.

What do you mean by conspiracy? Are the conservation laws "conspiracies" of this sort or what?

If you think there's some kind of conspiracy, explain why you think that.

Because it is a possible way to explain the EPR experiments. I see no reason to dismiss this hypothesis in favor of non-local theories.

Otherwise, the rational response is the same as if you had said "maybe we're all brains in vats" -- namely, unless you can provide at least some shred of evidence to suggest that this is possible, then it's just made up empty BS.

It is certainly possible. Can you provide evidence for a non-local mechanism? What exactly is your proposed non-local mechanism involved and why is it more believable?

I don't think he was wrong when he said that. I just don't think there is any actual evidentiary basis for believing that there is this kind of conspiracy.

The EPR experiments provide such an "evidentiary basis". You assume that your assumption is the default one when in fact it is barely tenable today. Tell me, what is the source for this free will? QM, right? So you must assume that QM is fundamentally non-deterministic in order to prove what you want. But this means that you reject a-priory all possible deterministic theories. Bell's theorem is just a red herring.

And by the way, there is a mountain of positive empirical evidence that this is *not* the case -- namely that we are able to understand a zillion other physical phenomena without any such massive metaphysical conspiracy.

Do you mean that you understand, really understand QM? That's great, enlighten me, please.

Note in particular that one could make this same objection to anything. A "randomized" double-blind drug trial showed that some new drug cures the cancer in all the patients that were given it? So this drug is the cure for cancer! Sweet! Oh, but no, you say that maybe it wasn't double-blind and randomized after all, there was some conspiracy that resulted in all the patients who would have survived *anyway* being just the ones who were "randomly" given the drug, so really the data is based on biased sampling and is invalid.

1. You keep using the word "conspiracy". I propose nothing of this sort. The EPR experiment itself shows us that there are "conspiracies". The question is how can we understand them.

2. Your "argument" reminds me of an argument for the existence of god. If there is no god then the evil is not punished, and the good is not rewarded. But we don't like that, do we? Therefore god exists.
Even if your premise (that determinism implies that the scientific method is flawed) is true (which is not) this doesn't prove that determinism is false.

3. If a drug is good in tests, it will probably be good in the future too. You see, I do not deny that QM correlations are true and I do not reject inductive reasoning. I believe that a perfect experiment (100% detector efficiencies, no noise, perfect source, no single detections and so on) will confirm QM's prediction. My position is that the explanation resides in the interaction between the source and detectors (including any randomizer, human experimenter and so on) and not in non-local effects.

Well, the fact is, in science, unless you can actually provide some kind of empirical basis for this speculation (even if it is minimal) nobody will or should listen to you.

That's easy. There is no scientific basis for free will. There is good scientific basis for locality (relativity). Bell's theorem requires us to drop one of these assumptions. I think free will should be the first to go.

And the proof that that's the right response is in the pudding: if people did listen to this kind of arbitrary made up objection, we wouldn't have the drugs we have to cure us of terrible diseases, or any of the other wonderful practical things that science has given us.

This conclusion does not follow from your premises at all.

Here is another objection you might worry about. Maybe all the people who claim to have done these experiments to test the Bell Inequalities have actually been bought off by the government to lie, in their published papers, about what the results were. It's possible, right? So the burden of proof is on you to prove that this didn't happen if you want to accept Aspect's published paper as really reporting the actual results of his experiments... right?

Straw man.

Um, actually that isn't possible. You can let the detectors affect the particle source all you want and it doesn't matter. The "complete state description" lambda that shows up in Bell's definition of locality is taken after the particles are created and in-flight toward the detectors. So either you have to let the detectors affect the state of the particles *after* they are in-flight (and this means that the detector on one side has to be able to affect the particle on the *other* side, which, the way the experiments are done, requires superluminal / nonlocal causation)... or you have to say that there is some kind of sample-biasing conspiracy whereby the "random" setting of the detector angle *after* the particles have already been emitting, actually isn't random at all, and in particular is correlated with the "random" setting on the far side and also with the state of the particle pair.

The field generated by a detector which doesn’t change its orientation is different from a mobile one. This is always true in a deterministic context where a certain state uniquely determines the future. Therefore, the source “knows” about the detector “intentions” long before the orientation is “chosen”. A good analogy is to be found in gravity. Earth “knows” the future position of the Sun and Moon (the tides point to the “instantaneous” position of those bodies and not towards their retarded ones (where we actually see them) and moves accordingly. But Earth does not communicate non-localy with the Sun, it only follows the local space curvature. I see no reason to dismiss that such a mechanism is responsible for EPR correlations.

You can see now that all the tricks in the world cannot destroy the correlations if the deterministic law is enforced at the microscopic level. Humans, computers, whatever devices you may think of, are just a bunch of charged particles that obey that deterministic law. The only way to falsify QM would be to produce the detectors out of nothing, with no history in this universe. But that’s just impossible.

So now it's not clear what your objection actually is. Before, in your other post to me, it sounded like you were hung up on the fair sampling issue, and the (arbitrary) possibility that one can't actually test the inequality in an unbiased way, because of some conspiracy of pre-determination in the detector settings.

That’s not my position.

But now it sounds like you think there is a much more serious flaw in Bell's argument -- that there are perfectly local mechanisms whereby the inequality could be violated. If you are worried about this, then you're just wrong and haven't understood the theorem.

There is no flaw, it’s a limitation of the theorem (a big one, indeed). The non-locality is introduced by the free-will assumption. And, I think I could make an even stronger claim here, that any deterministic theory looks non-local if free-will is assumed. For example, there is no way to explain locally the gravitational interaction if one assumes that the orbiting bodies have free-will. How could the Earth adjust its orbit if the Sun moves in an unpredictable way? Sure, there has to be some non-local communication between them. Yeah, right!
 
  • #136
DrChinese said:
Egads, I actually agree with this! LOL. Just to add: extending ueit's thinking, you could actually say that ALL scientific results show behavior that is false, because the results were "predetermined" to support an otherwise unsupportable conclusion.

That conclusion doesn't follow. Please read my answer to ttn.

I will point out this setup to demostrate how difficult ueit's position is:

Suppose the detector settings are determined by separate radioactive samples* (i.e. not an electromagnetic force) which are themselves spacelike separated from both the detector and the entanglement source. Thus the deterministic "first cause" is being propagated through space-time affecting even the splitting of atoms in a sample in such a way as to act in concert with both a distant photon source AND a second distant radioactive source to produce results that match the predictions of QM. Further, the example could be extended to involve dozens of separate radioactive sources, which must now operate in concert.

Your example is similar to that of an astronaut left in space, trying to change his center of mass and his orbit by moving his hands and legs in a very complex way. You know what? It ain't going to happen.

This doesn't necessarily mean that any deterministic theory is unfalsifiable, only that the method you propose cannot be used in this case.

AFAIK the weak and EM force have been unified so it doesn't seem so absurd that there is a connection between a radioactive decay and a EM process.

Why do you believe that such a connection is less likely than a non-local mechanism?
 
  • #137
ueit said:
This is not about empirical tests. The validity of the theorem itself depends on it.

I don't agree. The theorem just assumes that the "choices" (or, to use more neutral language, "selection" or whatever) of the settings on the two sides are uncorrelated with the state of the pair that's produced.

Whether that assumption is or is not realized in the experiments, is indeed "about empirical tests". It's not the validity of the theorem, but its applicability to the actual experimental tests, which is the question here.






If the detector orientations are not independent variables there is no Bell theorem. I'm not saying that the assumption is necessarily false in our universe (it might be that we have free will after all) but it is incompatible with the assumption of a deterministic HV theory underlying QM. In order to avoid a useless circular reasoning you have to drop this assumption when discussing deterministic theories therefore you cannot use Bell's theorem to dismiss such theories.

I think you misunderstand the relevance of "free will" here. None of the actual experiments actually have free will choices of the settings, and Bell doesn't in any way rely on free will *in particular*. The issue of free will only comes up as a kind of extreme thought-experiment to show that, in principle (well, unless you don't believe in free will), the settings on the two sides *could* be made in such a way that they'd be guaranteed to be uncorrelated with the particle pair's state. But as I said above, this doesn't really matter. All that's actually required is that the real experiments make the settings in a way that is sufficiently pseudo-random that there are no correlations between the settings and the particle states. There is every reason in the world to think this is realized. Let me repeat that: there is every reason in the world to believe that, in the actual experiments, the settings are not correlated with the particle states. The basic reason is clear: the experimentalists (who are aware of this possible loophole) go out of their way to have the settings be made independently and as randomly as anything can reasonably be made.

So, to repeat a point I made before, if someone wants to deny this -- if someone thinks that there *are* these correlations (which btw is what I meant by the word "conspiracy") -- the burden is squarely on them to provide some kind of argument or evidence. (And no, just saying "well maybe the world is deterministic, such that, in principle, anything that happens anywhere might be correlated with any other thing happenings somewhere else, by a local mechanism in the overlap of their past light cones" isn't good enough. That's just an empty, arbitrary, made up "possibility" which, in science, is treated like what it is: empty BS.)


Because it is a possible way to explain the EPR experiments. I see no reason to dismiss this hypothesis in favor of non-local theories.

Saying "maybe there was a conspiracy by which the settings [which the experimenters *tried* to make independent and random] were actually not at all independent and not at all random" is not an *explanation* of anything.

It's the same as the randomized drug trial example I gave the other day. If the double blind study shows empirically that 90% of the patients who get a certain drug subsequently recover from cancer, one (obvious) explanation of this fact is that the drug is curing (or helping cure) the cancer. You can't just say "maybe there was some accident or conspiracy whereby those patients who would have survived anyway, were the ones who got the drug, so really the drug doesn't help anybody." I mean, you can *say* it, but surely nobody thinks that's a genuine scientific *explanation* of anything. In science, properly, if you did say such a thing, the scientists would ask you: what's your evidence that there is some such accident/conspiracy? Do you have evidence that the double-blind character of the study was compromised? Do you have evidence that one of the doctors performing the trial is heavily invested in the stock of the company that makes the drug? etc... The point is, if all you have is an empty "maybe" and not any actual positive evidence, then you don't have anything.



Can you provide evidence for a non-local mechanism?

Um, yes, the empirical violation of Bell's inequalities.


What exactly is your proposed non-local mechanism involved and why is it more believable?

More believable than what? You haven't (nor has anyone else) offered any alternative. You've just said: maybe there's a problem with the experiments -- of a sort which, if really there, would indeed undermine my claim that the experiments show nonlocality... but since the possible biasing effects of that kind of problem are well known and are *deliberately* removed in the design of the experiments, there is no evidence (until or unless you provide some) that there actually is any such problem.


Tell me, what is the source for this free will? QM, right?

As I explained, nothing here actually relies on free will. And, on a totally unrelated note, I don't think QM (and its stochastic as opposed to deterministic character) is any kind of basis for belief in free will, at least not what *I* mean by "free will". Free will means we have some actual *control* over what we think and do -- not that we are "stochastic" in the sense of orthodox QM. OQM might support *indeterminism*, but it does not support free will.


1. You keep using the word "conspiracy". I propose nothing of this sort.

Yes, you do. You propose that the state of the emitted particles is affected by or affects (by a completely mysterious, unspecified mechanism) the polarizer settings on the two sides (or that they are just all magically correlated without any causal mechanism). That's a conspiracy.



3. If a drug is good in tests, it will probably be good in the future too. You see, I do not deny that QM correlations are true and I do not reject inductive reasoning.

You apparently missed the point of that example. It had nothing to do with the truth of QM's predictions or the validity of induction. It had to do with the *arbitrariness* of saying "maybe there's a conspiracy in the experiment which renders the otherwise-obvious interpretation invalid". When experimenters deliberately go out of their way to remove the possibility of a certain kind of correlation (as for example by making a study double-blind), the burden is on the person saying that kind of thing to actually produce some evidence. Otherwise, it's an empty maybe... i.e., it's *nothing*. It's as if you said "maybe magical fairies sprinkled pixie dust in the journal just before you came and looked at it, and if only you could see the real thing you'd find that the experiment had a very different outcome than the one reported there on the pages in front of you." Yeah, maybe.


I believe that a perfect experiment (100% detector efficiencies, no noise, perfect source, no single detections and so on) will confirm QM's prediction. My position is that the explanation resides in the interaction between the source and detectors (including any randomizer, human experimenter and so on) and not in non-local effects.


What "interaction between the source and detectors"? Don't you understand that the experimenters go out of their way to ensure that there is no such interaction (other than the source shooting particles toward the detectors, which they detect), since they know that could bias the data? For example, that's why they do the whole "delayed choice" thing -- without that, there could be a *local* mechanism by which the source "learns" about the settings, and then cleverly adjusts the state of the emitted particles in accordance with those settings, in order to make it *look* like Bell's inequalities are violated (where in fact, they aren't). And, not only do they make the settings at the last minute (after the particles have been created so their state is fixed), they make them *randomly* (i.e., in a way that is deliberately not correlated with anything having anything to do with the particle source) and not just that, but they make them randomly with *independent* random number generators on the two sides. So, unless you can say something about the particular "interaction between the source and detectors" that you have in mind, there is no basis for saying "maybe there is one, and it's biasing the results" -- and hence nothing further to talk about.
 
  • #138
DrChinese said:
Of course the realistic position is that P(a,c), P(a,b), and P(b,c) don't change according to your choice of which of these you choose to measure.

There you go with "realism" again. The correlations here -- P(a,b) or whatever -- also "don't change according to your choice of which of these you choose to measure" according to OQM. OQM just makes a certain prediction for what the correlations are for any two angles. It makes that prediction no matter what you or anybody measures. In particular, it makes the prediction it makes for P(a,b) and for P(a,c) regardless of which of these (or maybe something else entirely) someone chooses to measure.

So maybe you also think OQM is a "realist" theory? It wouldn't surprise me at this point since your use of that term is so slippery and vague.


And of course there is the assumption that:

1 >=P(a, b, c) >=0

which is also the realistic position restated.

Oh, so now "realism" means that probabilities have to be between zero and one? Gee, I always thought that was just part of the definition of what it meant to be a probability.

Or maybe you meant to highlight that it's P(a,b,c) specifically which has to be between zero and one? That it's somehow part of "realism" that this particular probability exist? Well what is this probability? Is it what I said before when I used that exact symbolism, namely: the correlation function for 3 simultaneous measurements of 3 spin directions on 2 particles? Because that is *meaningless*, as I explained before.



Who really cares if P(a, b) and P(a, c) can be measured separately UNLESS measurements at a, b and c all had simultaneous definite outcomes? Unless, as you point out, there is non-local communication occurring which skews the results...

Huh?

None of this response makes any sense at all. And it doesn't even touch on the essential points I made in my last post in response to you. I guess this is just your way of saying *something* (so it doesn't look like I stunned you into silence by blasting your myriad confusions to hell). OK, good job, you said something. Now maybe go back and do some homework on EPR and Bell's Theorem so you can figure out what you're talking about... before I spend any more time trying to talk to you about this stuff.
 
  • #139
RandallB said:
Fair enough, I think I at least understand where you’re at.

Good.

Maybe I'm wrong about Dr C, but unless I missed where he plainly declared BM as being local which would not be correct, I find no fault with his understanding of the Bell Theorem.

No, he doesn't think BM is local. But then, that doesn't/wouldn't have anything to do with Bell's theorem anyway. BM provided the motivation for Bell's discovery of the theorem, but the theorem (viz, that any viable theory must be nonlocal) stands independent of that or any other particular theory. BM is merely an example of a viable theory which is nonlocal.

As to Dr C's faulty understanding of Bell's Theorem, his confusion lies in thinking that there is some additional assumption (which he calls "realism", but who can figure out what he even means by this?) beyond Locality which is needed to derive a Bell Inequality, and hence which might be jettisoned (*instead* of locality) in the face of the experimental data. But even that is a bit generous, since, as his recent posts here show, he really just doesn't understand the theorem *at all*, and fundamentally misunderstands (as far as I can tell) all of the equations in Bell's papers. So his confusion about "realism" isn't even a confusion based on some kind of subtle misreading or misunderstanding of some aspect of Bell's proof (which by comparison would be a virtue). It's just a re-warmed re-hash of the sloppy ideas he's heard elsewhere, from other people who don't take the time to actually understand Bell correctly and first-handedly.


I’ll just leave it that I do not see the detail in your disputes that justify the abusiveness in some of your disagreements and it may well be beyond me.

That the abusiveness is uncalled for is probably a fair point. But there is a history (to put it in the most neutral possible terms) of us not understanding each other, despite endless cycles of clarification and repitition. So the abusiveness is born of frustration, and I suspect we both fail to self-censor because, at this point, neither one of us cares if our abusiveness makes the other stop talking to us. AT this point, the discussion is just "show" for other readers, so whether the other person is offended is irrelevant.


I do agree that in science the MAIN battle is between among several non-local theories, But, I do feel it is right and proper that at least a skirmish of a fight should continue in the search of HVT.

You make it sound like it's either/or: either nonlocal theories, or hidden variable theories. But that doesn't exactly cover the field. There are nonlocal HVTs (e.g., Bohm) which have a lot going for them.
 
  • #140
RandallB said:
? Reread post #97, billiard balls following Newton’s laws is Bell Local. Newton’s Laws do not require complete predestined and predetermined events. Newton does not deny the ability of an unknown ball with an unknown history from entering a local system – complete deterministic theories do, that’s why they demand no free will.

There can be no free-will in a purely Newtonian world. It's a contradiction in terms. Sure, you can sometime apply the theory without its conflict with free will to become obvious, just like in many QM experiments before EPR. But sooner or later the incompatibility must appear. A half-deterministic theory of the universe is a logical absurdity.
 

Similar threads

Replies
5
Views
964
  • Quantum Physics
2
Replies
36
Views
1K
Replies
19
Views
950
Replies
7
Views
1K
  • Quantum Physics
2
Replies
49
Views
3K
Replies
1
Views
636
  • Quantum Physics
Replies
4
Views
788
  • Quantum Physics
Replies
18
Views
1K
Replies
3
Views
769
  • Quantum Physics
Replies
14
Views
1K
Back
Top